Practice Test Mess Ups

Lakukan tugas rumah & ujian kamu dengan baik sekarang menggunakan Quizwiz!

A lawyer is the head of the in-house law department of a children's clothing company, which has its principal place of business within the state. Under state law, it is a felony to manufacture or sell children's sleepwear that is not fire retardant. The president of the company informed the lawyer in confidence that the company is stuck with a whole warehouse full of children's pajama fabric that does not meet the state's fire standards, and that to avoid financial disaster, the company will use the fabric to make children's sleepwear and take its chances on legal liability. The lawyer was unable to convince the president to change his mind; she then raised the issue with the company's board of directors, which ratified the president's decision. Will the lawyer be subject to discipline if she resigns as house counsel and reports the matter to the appropriate state law enforcement authorities? A Yes, because the lawyer is required to preserve the corporation's confidential information even after she resigns. B Yes, because there is no adequate reason for permissive withdrawal on these facts. C No, because the lawyer is entitled to reveal this type of confidential information. D No, because the lawyer's duty to preserve confidential information ceases with her resignation as house counsel.

The lawyer will not be subject to discipline for reporting the matter to the appropriate authorities because she is entitled to reveal this type of confidential information. The lawyer may resign her in-house counsel position because the company's board insists on following a course of action that is both repugnant and criminal. [ABA Model Rule 1.16(b)(2), (4)] The children's sleepwear is likely to cause substantial bodily harm or even death. Because an attorney is entitled to reveal confidential information to the extent she reasonably believes necessary to prevent reasonably certain death or substantial bodily harm, the lawyer may report this matter to the appropriate authorities. [ABA Model Rule 1.6(b)(1)] In addition, if the highest authority for an organization fails to take appropriate action regarding a violation of law, then a lawyer for the organization may report the relevant information to an appropriate person outside of the organization, if the lawyer reasonably believes that reporting is necessary to prevent substantial injury to the organization. This is true even if the information would otherwise be protected by the duty of confidentiality (which is not the case here because the sale of the fabric is likely to cause substantial bodily harm). [ABA Model Rule 1.13(c)] Here, the sale of the fabric is a violation of law, and selling the fabric could result in substantial injury to the organization if the fabric catches fire. (A) is wrong because the lawyer is entitled to reveal confidential information to prevent reasonably certain death or substantial bodily harm or to prevent substantial injury to the organization. (B) is wrong because the repugnance and criminality of the proposed conduct are both sufficient grounds for permissive withdrawal. [ABA Model Rule 1.16(2), (4)] (D) is wrong because the duty of confidentiality continues even after the termination of the attorney-client relationship. [See comment 18 to ABA Model Rule 1.6]

A lawyer was employed to represent a man at his trial for treason; the man was charged with smuggling top secret military information to a foreign government. The lawyer had reason to suspect that some of the prosecution's witnesses were paid liars. Therefore, with his client's consent, the lawyer hired to assist him in the defense a psychologist whose specialty is the behavior of liars. The client consented to the lawyer's advancing the psychologist's fee as a part of the expenses of litigation. At the trial, the psychologist sat with the lawyer at the counsel table. She watched the witnesses testify, and she advised the lawyer when she believed that a witness was lying and ought to be pursued on cross-examination. Most of the time, the lawyer followed the psychologist's advice, but sometimes he did not. Ultimately, the client was convicted and sent to prison for 20 years. Which of the following propositions is correct? Press Enter or Space to submit the answer A The lawyer is subject to discipline for allowing a third party to interject herself into the relationship between him and his client. B The lawyer is subject to discipline for advancing the psychologist's fee as a part of the expenses of litigation. C The lawyer is subject to discipline for allowing a nonlawyer to sit at the counsel table and to participate in his legal representation of a criminal defendant. D The lawyer is not subject to discipline because he did not give up his discretion to the psychologist, as shown by the fact that he did not always follow her advice.

(A) reflects a misconception of ABA Model Rule 1.8(f), which prohibits a lawyer from allowing the interests of a third party to interfere with the relationship between the lawyer and the client. That is not the case where an alleged expert like the psychologist is hired to assist the lawyer in representing the client. (C) reflects a similar misconception. Attorneys frequently use a variety of experts to advise them during trial. The lawyer did not give up his discretion to the psychologist, as evidenced by the fact that he sometimes did not follow her advice. Furthermore, such experts frequently sit at the counsel table so they can be close to the lawyer during trial. (B) is incorrect because a lawyer is allowed to advance litigation expenses. [See ABA Model Rule 1.8(e)(1)] Although some lawyers might question the wisdom of spending money on a purported expert on liars, that would seem to be a matter best left to the discretion of the individual lawyer, and the Model Rules do not suggest otherwise. Thus, (D) is correct, and (A), (B), and (C) are incorrect.

An attorney knows that the statute of limitations on her client's claim has run. However, the statute of limitations is an affirmative defense that the defendant in the case would waive if she failed to plead it. The attorney's client is willing to incur the legal fees and court costs of filing the lawsuit, and understands the risks. What may the attorney do? Press Enter or Space to submit the answer A File the suit because her client is willing to incur the legal fees and court costs. B File the suit but inform the court that the statute of limitations has run. C Not file the suit unless her client consents to disclose to the court the fact that the statute of limitations has run. D Not file the suit as it is now a frivolous claim.

(D) is incorrect; the claim is not frivolous under ABA Model Rule 3.1. The statute of limitations merely destroys the remedy and not the right. Unless the defendant pleads the statute of limitations, the claim is valid. Thus, (A) is correct. (B) and (C) are incorrect; there is no duty to inform the court that the statute of limitations has run.

For many years a lawyer has done business transactions work for a wealthy client. The client was recently injured in an automobile crash, and she has asked the lawyer to represent her as plaintiff in an action against the driver who injured her. The lawyer has taken some business cases to trial, but he has never handled a personal injury case. The lawyer would like to help his client and also generate some income. Which of the following would be an improper way for him to do so? Press Enter or Space to submit the answer A Take the case and, with the client's consent, associate a co-counsel who is competent in the field of personal injury law. B Refer the client to a competent personal injury lawyer and charge that lawyer a $1,000 forwarding fee. C Refer the client to a competent personal injury lawyer and charge the client a reasonable sum for the time spent in making the referral. D Take the case and, with the client's consent, undertake additional research to bring himself up to speed in the field of personal injury law.

A "forwarding fee" is another term for a "referral fee," and payments for referrals are prohibited. [ABA Model Rule 7.2(b)] (A) is proper because a lawyer may take on a case that he is not competent to handle if he obtains his client's consent to associate a lawyer who is competent to handle it. [Comment 2 to ABA Model Rule 1.1] (C) is proper because a lawyer may refer her client to another lawyer who is competent to handle the case. Making a sound referral can take a significant amount of time, especially if the referring lawyer needs to research the backgrounds of several lawyers with whom she is not personally familiar. It is appropriate for the referring lawyer to charge her client for the time spent making the referral, subject of course to the general rule on reasonableness. [ABA Model Rule 1.5] (As a practical matter, however, many lawyers would not charge a regular client for making such a referral.) (D) is proper because a lawyer may take on a case that he is not competent to handle if he undertakes the study necessary to provide competent representation. [Comment 2 to ABA Model Rule 1.1]

Before a judge was elected to the bench, she and her law partner purchased a piece of property to be held in co-tenancy by the judge and her law partner. After the judge was elected to the bench, she agreed to pay her law partner an annual fee to manage the property because her own time would be severely limited by her judicial duties. The judge and her law partner meet every three months to discuss the status of the property. The law partner sometimes appears as an attorney in the judge's courtroom. Was it proper for the judge to make this arrangement with her law partner? Press Enter or Space to submit the answer A Yes, because the judge acquired the property before she became a judge. B Yes, because the judge made arrangements to ensure that her judicial duties would not suffer. C No, because judges should not engage in remunerative outside enterprises. D No, because the law partner appears in cases before the judge's court.

A judge should refrain from financial and business dealings that involve her in frequent transactions with lawyers or persons likely to come before the court on which she sits. [CJC Rule 3.11(C)(3)] Here, the judge's close business relationship with a lawyer who appears in her courtroom violates CJC Rule 3.11(C)(3). It makes no difference that the judge acquired the property before she became a judge. Thus, (A) is incorrect. (B) is incorrect because even if the judge's dealings with the property do not take up too much of her time, there will still be an adverse reflection on her impartiality. (C) is incorrect because CJC Rule 3.11(A) permits a judge to hold investments, including real estate, and to engage in other remunerative activity.

An attorney recently opened his solo law practice in a small town. His practice is fairly evenly divided between civil litigation and criminal defense. The Superior Court has just appointed the attorney to represent two defendants, who will be tried jointly for their alleged kidnapping and brutal murder of nine local school children. Which of the following is not a valid reason for the attorney to decline the appointment? A He believes that to represent the two defendants will take so much time away from his newly opened practice as to impose an unreasonable financial burden on him. B He believes that one defendant coerced the other defendant into helping kidnap and kill the children. C He believes many of his potential clients will be outraged if he represents the two defendants. D He believes that confidential information he received when representing one of the prosecution's key witnesses will be useful in impeaching that witness's credibility.

A lawyer can be disciplined for trying to avoid a court appointment without good cause. [ABA Model Rule 6.2] The reason stated in (C) is not an acceptable reason for declining the appointment; a lawyer has a duty to represent his fair share of indigent or unpopular clients. [Comment 1 to ABA Model Rule 6.2] (A) is a legitimate reason for declining an appointment. A lawyer is permitted to turn down a court appointment if it "is likely to result in an unreasonable financial burden." [ABA Model Rule 6.2(b)] (B) is also a legitimate basis for declining appointment because a lawyer may turn down a court appointment if it is likely to cause the lawyer to violate a rule of professional conduct. [ABA Model Rule 6.2(a)] If the two defendants are to be tried jointly, and if one defendant did coerce the other defendant into helping with the kidnapping and killing, there is a sharp conflict of interest between the two defendants. [See ABA Model Rule 1.7(a)] It would be an ethical violation to represent co-defendants with conflicting interests (consent will not solve the conflict); thus, the attorney can decline the appointment on this ground. (D) raises another conflict of interest that would justify the attorney in declining the appointment. Had he not gained confidential information from the prosecution's witness, he might have discovered that information independently and been able to use it to impeach the witness. As it stands, however, his ability to impeach is constrained by his duty not to use the confidential information to the disadvantage of the witness, his former client. [ABA Model Rule 1.9(c)(1)]

The law of a particular state prohibits agreements not to compete, except for agreements that are ancillary to the sale of a business or professional practice and are reasonable in both duration and geographic scope. For 20 years, an attorney practiced patent, copyright, and trademark law in an area within the state. Seeking a new challenge, the attorney entered the political race for a trial court judgeship. He won a four-year term. Before taking the oath of judicial office, the attorney sold his entire law practice to a young lawyer for $150,000. In the sale contract, the attorney promised the younger lawyer not to re-enter the practice of patent, copyright, or trademark law in the area for five years. At the end of his four-year term, the attorney ran for re-election to his judgeship; to everyone's great surprise, he lost the election to a much less qualified opponent. Because he needed to earn a living, the attorney immediately re-entered the practice of patent, copyright, and trademark law in the area in which he formerly had practiced. Is the attorney subject to discipline? Press Enter or Space to submit the answer A Yes, because the attorney re-entered law practice in the area after the sale to the younger lawyer. B Yes, because the attorney made an agreement that restricts his right to practice law and it was not incident to a retirement benefits plan. C No, but the attorney could be subject to civil liability to the younger lawyer in a suit to enforce the agreement not to compete, assuming that the agreement was reasonable in duration and geographic scope. D No, because the sale of the attorney's law practice was proper, but the agreement not to compete is void, even if it is reasonable in duration and geographic scope.

ABA Model Rule 1.17 permits a practicing lawyer to sell his entire law practice or an area of his law practice to one or more lawyers or law firms. That is what the attorney did here shortly before becoming a judge. One of the conditions specified in ABA Model Rule 1.17 is that the seller must cease "to engage in the private practice of law . . . in the [jurisdiction or geographic area] in which the practice has been conducted." [ABA Model Rule 1.17(a)] The attorney satisfied that condition by becoming a judge-engaging in judicial duties does not constitute the private practice of law, nor does working as a government lawyer, lawyer for the poor, or an in-house lawyer for a business. [See comments 2 and 3 to ABA Model Rule 1.17] The attorney's re-entry into private practice does not violate ABA Model Rule 1.17 because it was caused by an "unanticipated change in circumstances"-i.e., his failure to win re-election. [See comment 2 to ABA Model Rule 1.17, which uses re-election defeat as an example of "unanticipated change"] (A) is wrong for the reason just stated. (B) is wrong because ABA Model Rule 5.6 (the rule that prohibits agreements that restrict a lawyer's right to practice) does not apply to agreements that are ancillary to the sale of a law practice. [Comment 3 to ABA Model Rule 5.6] (D) is wrong because the law of the state makes the attorney's agreement not to compete valid if it is reasonable in duration and geographic scope.

A criminal defendant is facing trial for the unlawful possession of a large quantity of a controlled substance-specifically, 400 kilograms of cocaine. The accused originally asserted that he could not afford to pay a lawyer, and the court arranged for him to be represented by a public defender. Later, the accused was visited in jail by a man who identified himself as a skilled criminal defense lawyer who had been hired by the accused's "friends" to defend him. The man said that the accused would not have to pay anything for the legal defense if he "played the game straight" and did not implicate the "friends" in the cocaine caper. The accused consented to the conditions and to the third party payment of his legal fees. In due course, the man was substituted in as defense counsel. Shortly before trial, the prosecutor offered the accused an attractive plea bargain-a mere six months in jail in exchange for a guilty plea, an identification of the persons for whom the accused was transporting the cocaine, and testimony against those persons. The man solemnly advised the accused to reject the plea bargain, saying there would be dire consequences. The accused did reject the plea bargain. Thereupon the prosecutor moved to disqualify the man as defense counsel on the ground that he was being paid by an unidentified third party and that the third party was unduly interfering with the accused's constitutional right to effective assistance of counsel. Is the man subject to disqualification? Press Enter or Space to submit the answer A Yes, because in a criminal case it is unlawful for a private criminal defense lawyer to accept compensation from an outsider for his legal services. B Yes, because both the prosecutor and the judge have a duty to protect the defendant's constitutional right to the effective assistance of legal counsel. C No, because the accused consented to the fee arrangement after having been informed of the conditions on which it was offered. D No, because the prosecutor has no right to interfere with a criminal defendant's choice of counsel.

ABA Model Rule 1.8(f) prohibits a lawyer from accepting compensation for representing a client from anyone other than that client, unless two conditions are satisfied: (i) the client gives informed consent, and (ii) the person who pays the compensation does not interfere with the representation of the client or with the lawyer's independence. [See also Restatement §134, comment d] In this case, the "friends" are interfering with the lawyer's representation of the client and with his independence by conditioning their financial aid on the accused's not implicating them in the cocaine caper. The results of their interference become obvious when the accused is coerced into turning down the attractive plea bargain. [See Quintero v. United States, 33 F.3d 1133 (9th Cir. 1994)-similar facts] (A) and (C) are both incorrect because they misstate the principles expressed in ABA Model Rule 1.8(f)-(A) is too broad, and (C) does not take into account the interference by the "friends." (D) is incorrect because the prosecutor has an ethical obligation to help assure a criminal defendant's right to counsel. [See ABA Model Rule 3.8(b)]

A paralegal works for the law firm of Alpha & Beta. Her direct supervisor is partner Alpha, whose practice is limited to international trade law. Partner Beta is the firm's leading trial lawyer, both in commercial and personal injury cases. On her way to work one morning, the paralegal saw a pedestrian run down in a crosswalk by a speeding car. The paralegal rendered first aid, and while she was waiting with the pedestrian for the ambulance, the paralegal provided the pedestrian with a business card and urged him to call the firm to obtain legal representation in connection with his injuries. When she got to work, she told partner Alpha what she had done. Alpha admonished the paralegal not to hand out the firm's cards in such situations, but he did not discuss the matter with partner Beta. Is Alpha subject to discipline? A Yes, because he failed to warn Beta not to take the pedestrian's case. B Yes, because as the paralegal's supervisor, he is responsible for any unethical act she commits. C No, because as a nonlawyer, the paralegal is free to recommend a lawyer to someone if she wishes. D No, because the paralegal may not have been aware at the time that she did anything wrong.

Alpha is subject to discipline for failing to warn Beta not to take the case. If the paralegal were a lawyer, her conduct would violate ABA Model Rule 7.3(b), which prohibits in-person solicitation. The partners in a firm are responsible for educating their nonlawyer employees about ethics issues and making reasonable efforts to assure that those employees comply with ethics rules. [ABA Model Rule 5.3(a)] Moreover, a partner is subject to discipline if he learns about the violation of an ethics rule by a nonlawyer employee "when its consequences can be avoided or mitigated," but the partner "fails to take reasonable remedial action." In this case, the consequences of the paralegal's solicitation could have been avoided by warning Beta not to take the pedestrian's case. Because he failed to warn Beta, Alpha is subject to discipline. (B) is wrong because it is too broad. A lawyer's responsibility for a nonlawyer employee's ethics violation is limited to situations in which the lawyer orders it, ratifies it, or learns about it in time to remedy it and does not do so. [ABA Model Rule 5.3(c)] (C) is wrong because even though people are generally free to recommend a lawyer to someone else, that does not allow the paralegal to solicit business for the firm that employs her. [ABA Model Rule 8.4(a)] (D) is wrong because Alpha and the other partners in the firm had a duty to educate the paralegal about ethics rules. [ABA Model Rule 5.3(a)] Furthermore, even if the paralegal acted innocently, that does not excuse Alpha's failure to warn Beta not to take the case.

An attorney represented a father in a custody dispute with his ex-wife regarding their child. As a result of the custody proceeding, the father lost custody of his child. Subsequently, the father sued the attorney, asserting that the attorney failed to diligently represent him in the proceeding because she had taken on too many clients. At trial, the attorney seeks to disclose information she acquired from her client regarding his finances as evidence that he is suing because he is upset about the significant support payments he has to make. Is the attorney's disclosure of the financial information she acquired from her client proper? No, because her client's finances are not relevant to the dispute. No, because the information was acquired during the course of the representation. Yes, because a dispute between an attorney and a client allows for the disclosure of confidential communications. Yes, because the client, by filing suit against his attorney, consented to the disclosure of confidential information acquired by the attorney during the representation.

Answer choice A is correct. A dispute between an attorney and her client (e.g., a malpractice allegation) or between co-clients who are now adverse to each other allows for the disclosure of confidential communications relevant to the dispute. The attorney could disclose information gained during her discussions with the client that was relevant to his allegations against her. Financial information is not relevant to the dispute because the client never claimed there was any issue with attorney's fees or any other financial aspect of the case; the attorney should be able to prove that she was diligent without disclosing her client's financial information. Answer choice B is incorrect because the attorney might be able to disclose this information if it was relevant to the dispute. Answer choice C is incorrect because, while such a dispute does allow for the disclosure of certain confidential information, it only allows for the disclosure of information that is relevant to the dispute. Answer choice D is incorrect because, as noted with regard to answer choice C, the client has not consented to the disclosure of any information beyond that relevant to the dispute.

A law firm, a professional corporation with five lawyer shareholders, employs twenty-five additional lawyers. Which of the following is proper? Answers: A. Employees who are members of the bar are not made shareholders until they have been with the law firm ten years. B. An office manager, who is not a member of the bar, is executive vice president of the law firm. C. A widow, whose husband was a lawyer shareholder in the law firm until his death two years ago, continues to hold her husband's shares in the law firm, distributed in his estate, until their child completes a law school education. D. For the first two years of employment, new associates of the law firm must have all of their work approved by a supervisor who is not a lawyer.

Answer choice A is correct. A law firm is permitted to set its own rules regarding membership in the firm by lawyers. Answer choice B is incorrect, since under MRPC 5.4(d), a lawyer is not permitted to practice with or in the form of a professional corporation if a non-lawyer is an officer thereof. Answer choice C is incorrect, as the widow, a non-lawyer, may hold the shares in the professional corporation only for a reasonable period of time during administration, not until her child has completed law school. Answer choice D is incorrect, as it is not permissible for a non-lawyer to direct or control the professional judgment of a lawyer practicing in a professional corporation. MRPC 5.4(d).

A defendant is charged with robbery. A police officer received an anonymous tip that a specific individual other than the defendant committed the robbery. The police officer investigated the information, but concluded that the other individual did not commit the robbery. The police officer conveyed this information to the prosecutor. Due to the existence of other evidence that linked the defendant to the robbery, the prosecutor concluded in good faith that the tip, while exculpatory, was not material, and therefore she did not have a constitutional duty to turn the information over to the defense. Consequently, the prosecutor did not reveal the tip to the defendant's lawyer, who failed to make a Brady request for exculpatory evidence. State criminal procedure discovery rules did not require the prosecutor to disclose the anonymous tip. Is the prosecutor subject to discipline for her failure to inform the defense of the anonymous tip? Yes, because the prosecutor determined that the anonymous tip was exculpatory information. Yes, because a prosecutor has a duty to search for exculpatory information. No, because the defendant's lawyer did not make a Brady request for exculpatory evidence. No, because the prosecutor acted in good faith in determining that the anonymous tip did not have to be turned over to the defense.

Answer choice A is correct. A prosecutor has an ethical duty to make timely disclosure to the defense of all evidence or information known to the prosecutor that tends to negate the guilt of the accused or mitigates the offense. MRPC 3.8(d). Since the prosecutor concluded in good faith that the anonymous tip was exculpatory, the prosecutor had a duty to disclose the tip to the defense. Answer choice B is incorrect because, while a prosecutor does have a duty to reveal exculpatory information of which the prosecutor has knowledge, a prosecutor is not under a duty to search for such information. Answer choice C is incorrect because, under MRPC 3.8(d), the prosecutor has a duty to reveal exculpatory information regardless of whether the defense requests such information. Answer choice D is incorrect because the prosecutor's good faith evaluation is part of her constitutional disclosure obligation. The ethical duty imposed by MRPC 3.8(d) is broader than the constitutional obligation. As noted with respect to answer choice A, since the prosecutor determined that the tip was exculpatory, the prosecutor was required by MRPC 3.8(d) to disclose the tip to the defense.

A prominent politician contacted a well-respected and successful lawyer about representing her in a divorce action. During the initial consultation, the politician revealed that she was having an affair. At the conclusion of the consultation, the lawyer declined to represent her. The politician responded, "Good. I only wanted to make sure you couldn't represent my husband." Shortly afterwards, the lawyer, in order to discredit the politician, revealed to a reporter that the politician was having an affair. Is the lawyer subject to discipline? Answers: A. No, because the lawyer did not owe a duty of confidentiality to the politician. B. No, because the lawyer declined to represent the politician. C. Yes, because the lawyer revealed information learned from a consultation with a prospective client. D. Yes, because the lawyer revealed the politician's affair in order to discredit her.

Answer choice A is correct. Although a lawyer owes a duty of confidentiality to a prospective client, a person who contacts a lawyer for the purpose of preventing the lawyer from representing another party in a matter (i.e., to disqualify the lawyer) is not treated as a prospective client. Consequently, the lawyer did not owe a duty of confidentiality with regard to the information about the politician's affair. Answer choice B is incorrect because the duty of confidentiality with regard to information learned during an interview with a prospective client about representation of the prospective client applies, even though the lawyer does not accept the representation. Answer choice C is incorrect because, as noted with regard to answer choice A, although a lawyer owes a duty of confidentiality to a prospective client, a person who contacts a lawyer for the purpose of preventing the lawyer from representing another party in a matter (i.e., to disqualify the lawyer) is not treated as a prospective client. Answer choice D is incorrect. Although the duty of confidentiality generally applies to the disclosure of any information learned during a consultation with a potential client, and it includes information used to the disadvantage of a client, the duty does not apply when a person contacts a lawyer for the purpose of preventing the lawyer from representing another party in a matter (i.e., to disqualify the lawyer).

A partner in a law firm was a witness to an assault that occurred in his neighborhood. The partner will likely have to testify in the civil action brought by the victim of the assault for damages. An associate at the partner's firm has agreed to represent the victim in the assault action. Is the associate subject to disqualification? Answers: A. No, because there is no conflict of interest. B. No, because the action is not a criminal case. C. Yes, because the partner is likely to be called as a witness. D. Yes, because the possibility of the partner being called as a witness creates a conflict that is imputed to the associate.

Answer choice A is correct. An attorney is permitted to act as an advocate in a trial in which another attorney in his firm is likely to be called as a witness, unless the conflict-of-interest rules otherwise preclude the attorney from doing so. Here, the facts do not indicate that there is any conflict, and the fact that the partner may be called as a witness does not prohibit a member of his firm from representing that client. Answer choice B is incorrect because the advocate-witness rule applies to civil as well as criminal actions. Answer choice C is incorrect because, unless some other conflict would prevent the associate from representing the client, the likelihood of the partner being called as a witness would not prohibit the associate from serving as the client's advocate. Answer choice D is incorrect because absent a conflict of interest regarding another matter, the partner's disqualification from serving as an attorney for the victim is not imputed to an attorney who is a member of the partner's law firm.

During the trial of an age discrimination lawsuit, both the judge and the attorney referenced the plaintiff's age and political affiliation multiple times in closing argument and jury instructions. The judge stated that "while members of an older generation may be more sensitive to being passed over for promotions, the law does not allow for actual discrimination." He also stated that individuals of the plaintiff's political party might face additional challenges in the particular field, which tends to be composed mostly of members of an opposing political party, and that this fact may have made the plaintiff feel alienated by his peers. Is it proper for the judge to reference the plaintiff's age and political affiliation in the above-referenced manner? Answers: A. No, as to both age and political affiliation B. Yes, as to both age and political affiliation C. Yes, as to age only D. Yes, as to political affiliation only

Answer choice A is correct. In performing judicial duties, a judge must not by words or conduct manifest bias or prejudice. Examples of manifestations of bias or prejudice include, but are not limited to: negative stereotyping; attempted humor based upon stereotypes; suggestions of connections between race, ethnicity, or nationality and crime; and irrelevant references to personal characteristics. The requirements of the Code of Judicial Conduct do not preclude judges or attorneys from making legitimate reference to race, sex, gender, religion, national origin, ethnicity, disability, age, sexual orientation, marital status, socioeconomic status, or political affiliation, or similar factors, when they are relevant to an issue in a proceeding. Here, the judge's reference to age reflected negative stereotyping, and was not permitted. Political affiliation was not relevant at all to an age discrimination issue, so the reference was not a legitimate exercise of judicial authority. Answer choice B is incorrect because the judge's comments for both age and political affiliation were improper. Answer choice C is incorrect because the judge's comments about age reflected inappropriate stereotyping. Answer choice D is incorrect because the judge should not have referenced the plaintiff's political affiliation, which was not a relevant issue.

An attorney represented a plaintiff who sued a defendant for injuries the plaintiff sustained in a car accident. Prior to trial, the attorney interviewed a witness who stated that she had observed the defendant drinking heavily hours before the accident. Unfortunately, on the eve of trial, the witness informed the attorney that she was ill and could not testify at trial. The attorney tried but could not obtain a continuance. As a result, the plaintiff's direct case rested solely on the plaintiff's testimony that the defendant was speeding and that the defendant's car crossed the middle line and hit the plaintiff's car. The defendant testified that he was driving safely in compliance with all rules and that the accident was entirely the plaintiff's fault. On cross-examination, the attorney asked the defendant, "Isn't it a fact that you were drinking prior to the accident?" The defendant answered that he had not consumed alcoholic beverages on the day of the accident. In summation to the jury, the attorney stated: "Ladies and gentlemen of the jury, you and I know that the defendant lied when he stated that he had not consumed alcoholic beverages on the day of the accident. We know that he was impaired." On which of the following grounds, if any, would the attorney NOT be subject to discipline? Answers: A The attorney's question to the defendant implying that the defendant had consumed alcoholic beverages when the attorney knew that he could not offer evidence of the defendant's drinking. B The attorney's statement to the jury asserting that the attorney knew that the defendant was drunk when no evidence in the record supported this allegation. C The attorney's statement asserting a personal belief that the defendant was drunk and lying. D The attorney is subject to discipline on all of these grounds.

Answer choice A is correct. This statement would not subject the attorney to discipline. The attorney was impeaching the defendant on cross-examination with knowledge of what the witness had told the attorney, and it is therefore permissible to imply that the defendant had been drinking, even if the attorney could not produce the witness to rebut a denial by the defendant. Answer choice B would subject the attorney to discipline, as MRPC 3.4(e) prohibits a lawyer in a trial from alluding to any matter that is not supported by admissible evidence. Answer choice C would subject the attorney to discipline, as a lawyer is prohibited from stating a personal opinion as to the justness of a cause, the credibility of a witness, or the guilt or innocence of an accused. MRPC 3.4(e).

An attorney, who had represented a testator for many years, prepared the testator's will and acted as one of the two subscribing witnesses to its execution. The testator's sister and brother were his sole heirs. The will left the testator's entire estate to his sister and nothing to his brother. Upon the testator's death two years later, the executor named in the will asked the attorney to act as his lawyer in the probate of the will and the administration of the estate. At that time, the executor informed the attorney that the testator's brother would concede that the will was properly executed but intended to contest the will on the ground that he had been excluded because of fraud previously practiced on the testator by the testator's sister. The other subscribing witness to the will predeceased the testator, and the attorney will be called as a witness solely for the purpose of establishing the due execution of the will. Is it proper for the attorney to accept the representation? Answers: A Yes, if there is no contested issue of fact with respect to the formal execution of the will. B Yes, because the executor has no beneficial interest under the will. C No, unless the attorney's services are necessary to avoid substantial hardship to the executor. D No, because the attorney will be called as a witness in the case.

Answer choice A is correct. Under MRPC 3.7(a), a lawyer is permitted to act as an advocate at a trial in which the lawyer is likely to be a necessary witness if the testimony relates to an uncontested issue. If there is no contested issue of fact with respect to the formal execution of the will, then it is proper for the attorney to accept the representation. Answer choice B is not correct, as it is irrelevant that the executor has no beneficial interest under the will. A lawyer is only permitted to act as an advocate at a trial in which the lawyer is likely to be a necessary witness if (i) the testimony relates to an uncontested issue, (ii) the testimony relates to the nature and value of legal services rendered in the case, or (iii) disqualification of the lawyer would work substantial hardship on the client. MRPC 3.7(a). Answer choice C is not correct. Even though avoiding substantial hardship to the client is a ground for the attorney to properly represent the executor under MRPC 3.7(a), it is not the only ground, which the phrasing of this answer choice suggests. Answer choice D is not correct, as MRPC 3.7(a) does allow a lawyer to be a witness and act as an advocate when the testimony relates to an uncontested issue.

An attorney and a prospective client met to discuss whether the attorney would represent the client in a contractual dispute. During the conversation, the potential plaintiff spoke to the attorney about her litigation objectives and how much she would be able to pay the attorney. As they were wrapping up the meeting, the client noticed a picture of the attorney's teenaged son on the wall. The client confided in the attorney that she had a son the same age, but she had given him up for adoption because she was an unwed teenager when he was born. She told the attorney that no one except her family knew about the adoption, and she asked the attorney to keep it confidential. Is the information about the client's pregnancy protected by the attorney-client privilege? Answers: A No, because the woman had not retained the attorney when the conversation took place. B No, because the communication was not relayed for legal advice. C Yes, because the woman had a reasonable expectation that she had established an attorney-client relationship with the attorney. D Yes, because the woman had a reasonable expectation that she had established an attorney-client relationship with the attorney, and she stated that the communication was confidential.

Answer choice B is correct. A confidential communication between a client and her attorney is privileged. The evidentiary attorney-client privilege covers the client's communication to an attorney whom the client reasonably believes represents the client, and the circumstances indicate a desire by the client for confidentiality. Although the attorney does not need to give advice or agree to the representation for the privilege to exist, the communication must be for the purpose of seeking legal advice or representation. Although the statement about the client's child was given in confidence and would be protected by the attorney's ethical duty of confidentiality, it is not subject to the attorney-client privilege because it was not relayed for the purpose of obtaining legal advice. Answer choice A is incorrect because, although the woman was a potential client with regard to the contract matter, the privilege would not apply to the statement regarding the adoption because the statement was not made for the purpose of obtaining legal advice. Answer choice C is incorrect because the communication was not made for the purpose of obtaining legal advice, so it is not privileged even if the woman had a reasonable expectation of representation. Answer choice D is incorrect because, although the communication would have been privileged if it had been relayed for the purpose of obtaining legal advice, it was not relayed for that purpose.

A plaintiff filed a personal injury complaint, and the case was assigned to a judge. After the defendant was served, a partner from a large law firm filed an appearance on behalf of the defendant. The judge's niece was a salaried associate in the estate planning department of the law firm representing the plaintiff. At the initial scheduling conference, the judge disclosed this relationship to the parties. Subsequently, the judge also disclosed that a person listed by the plaintiff as a material witness was his wife's nephew. Neither the niece nor the nephew resided in the judge's household. Neither party moved to disqualify the judge. Other than the disclosures made by the judge, there were no grounds upon which the judge's impartiality could be reasonably questioned. Should the judge disqualify himself from presiding over this action? Yes, because of the judge's relationship with a member of the law firm representing the defendant. Yes, because of the judge's familial relationship with the material witness. No, because neither party moved to disqualify the judge. No, because neither the niece nor the nephew were members of the judge's household.

Answer choice B is correct. A judge must disqualify himself in a matter if he knows that he or his spouse or domestic partner shares a third-degree or closer relationship to a material witness in a case. Since the nephew of the judge's wife falls within the third degree of relationship, the judge should recuse himself. Answer choice A is incorrect because a judge must disqualify himself in a matter if he knows that he or his spouse or domestic partner shares a third-degree or closer relationship to an attorney who represents a party in the case. A third-degree relationship includes a niece of the judge. However, the employment of the judge's niece as a lawyer by the same law firm as the attorney who represents a party does not automatically require the judge to disqualify himself unless the niece has more than a de minimis interest that could be affected by the proceedings. Since the niece was not employed by the department of the firm handling the case and since the income of the niece, as a salaried associate, would not be directly affected by the outcome of the case, the niece's interest in the case likely was de minimis. Accordingly, the judge's impartiality probably cannot be reasonably questioned on the grounds of his relationship with his niece, making B a better answer. Answer choice C is incorrect because a judge may be required to disqualify himself even if a party does not seek his disqualification. Answer choice D is incorrect because, although the standard for disqualification when a relative of the judge has an economic interest in the subject matter of the controversy or is a party to the proceeding is based in part on whether the relative resides in the judge's household, disqualification can be appropriate for a relative within the third degree of relationship even though that relative does not reside in the judge's household in certain circumstances, such as when a relative is likely to be a material witness.

An attorney decided to accept a case although a verdict in favor of his client would cause his own property to decrease substantially in value. The attorney explained the situation to the client, and the client consented in writing to the representation. After the court ruled against the attorney, the client filed a complaint with the disciplinary board alleging that the attorney should not have accepted the case. By what standard should the attorney's action be judged to determine whether he violated the conflict-of-interest rules? Answers: A The attorney must have honestly believed that he could provide competent and diligent representation to the client. B The attorney must have reasonably believed that he could provide competent and diligent representation to the client. C The client must have believed that the attorney could provide competent and diligent representation to him. D The attorney must have provided competent and diligent representation to the client.

Answer choice B is correct. An attorney must not represent a client if the representation of the client may be materially limited by the attorney's own interests, unless, in addition to securing the client's informed consent, confirmed in writing, the attorney reasonably believes that he will be able to provide competent and diligent representation to the affected client. The attorney's belief in his ability to provide competent and diligent representation is viewed under both a subjective and an objective standard. Answer choice A is incorrect because it is not enough for the attorney to have honestly believed that he could provide competent and diligent representation; such belief must also be reasonable. Answer choice C is incorrect because, although the client must give informed consent, confirmed in writing, it is the attorney who must possess a reasonable belief as to his ability to provide competent and diligent representation. Answer choice D is incorrect because, while the attorney may be disciplined for his failure to provide competent and diligent representation to his client, whether the attorney violated the conflict-of-interest rules turns on whether the attorney reasonably believed that he would be able to provide competent and diligent representation.

An attorney was engaged to create a trust for a client. The attorney, working to acquire an LL.M. in taxation, had taken several advanced tax law classes. The attorney did not advertise her advanced knowledge of the tax law because she had not been certified as a tax specialist by a state-approved or ABA-accredited organization, and the client was not aware that the attorney had any training in tax law. The trust created by the attorney ran afoul of tax provisions that a lawyer with her special knowledge of the tax law should have been able to avoid. As a consequence, the settlor incurred a tax liability that he otherwise would not have incurred. May the attorney be subject to civil liability to the settlor? Answers: A. Yes, because the attorney could not disclaim her possession of specialized knowledge of the tax law. B. Yes, because the attorney failed to exercise the special skill in tax law that she possessed. C. No, because the client was not aware and therefore did not rely on the attorney's special tax training. D. No, because the attorney did not hold herself out as a tax specialist.

Answer choice B is correct. In determining a lawyer's duty of care, the lawyer's special experience with or knowledge of the law is taken into account. Because the attorney should have been able to avoid the adverse tax consequences to the settlor by using her special knowledge of the tax law, the attorney has breached her duty of care and may be liable to the settlor for the tax liability incurred by the settlor. Answer choice A is incorrect because, at least in some circumstances, a lawyer may disclaim competency in excess of the ordinary or the possession of specialized knowledge. However, no disclaimer was made here. Answer choice C is incorrect because a lawyer has a duty to exercise any special skills or to apply special knowledge when acting on behalf of her client, unless she properly disclaims such skills or knowledge. Note: Such duty does not extend to a non-client who is not aware of the lawyer's skills or knowledge and who therefore does not rely on them. Answer choice D is incorrect because, although the attorney properly did not hold herself as a tax specialist, the attorney was required to use the specialized knowledge that she possessed.

An associate in a small law firm represented a property owner in a suit against the former owner, a corporation, for fraud with regard to the sale of the property. The associate has learned that the largest shareholder of a subsidiary of the corporation is the managing partner of the law firm. If the property owner is successful in the action, the corporation and its subsidiary would be crippled financially, and the managing partner would likely see a significant decline in the value of his stock, which represents a substantial portion of the managing partner's retirement portfolio. Must the associate share with the property owner the information regarding the managing partner's relationship to the corporation and seek the property owner's consent in order to continue representing him? Yes, because a personal disqualification of a member of a firm is imputed to all other members of the firm. Yes, because there is a significant risk of the managing partner's relationship to the opposing party materially limiting the associate's representation of the property owner. No, because the associate, not the managing partner, is representing the property owner. No, because the associate's duty of confidentiality prevents him from disclosing information about the partner's involvement in the corporation.

Answer choice B is correct. Members of a law firm are generally treated as one lawyer for purposes of the disqualification rules. When one member of the firm is disqualified from representing a client, all members of the firm are generally disqualified. However, there is an exception when the disqualification is based on a personal interest of a member of the firm and that interest does not present a significant risk of materially limiting the representation of the client by the remaining lawyers in the firm. Here, the managing partner would be personally disqualified from representing the property owner because of the managing partner's significant financial ties to the opposing party in the lawsuit (i.e., the corporation) and the significant risk that the associate would be materially limited in representing the property owner, out of loyalty to the partner. Answer choice A is incorrect because generally a disqualification based on a personal interest of a member of the firm is not imputed to other members of the firm if it does not present a significant risk of materially limiting the representation of the client by the remaining lawyers in the firm. Answer choice C is incorrect because members of a law firm are generally treated as one lawyer for purposes of the disqualification rules. In this case, the managing partner's disqualification is imputed to the associate as discussed with respect to answer choice B. Answer choice D is incorrect because the duty of confidentiality is owed by an attorney to a client. This duty does not apply to information regarding other members of the attorney's law firm.

A judge in a state criminal trial court wishes to serve as guardian of her father, who has been declared incompetent. Accepting the responsibilities of the position would not interfere with the performance of the judge's official duties. Although the position in all likelihood would not involve contested litigation, it would be necessary for the judge to prepare and sign various pleadings, motions, and other papers and to appear in civil court on her father's behalf. Would it be proper for the judge to undertake this guardianship? Answers: A Yes, unless the judge receives compensation for her services as guardian. B Yes, because the position involves a close family member and will not interfere with the judge's performance of her judicial duties. C No, because the position will require the judge to appear in court. D No, because the position will require the judge to prepare and sign pleadings, motions, and other papers.

Answer choice B is correct. Under CJC Rule 3.8(A), a judge may accept appointment to serve as a guardian for a member of the judge's family, but only if such service will not interfere with the proper performance of judicial duties. A judge may not serve in such position if the judge, as a fiduciary, will likely be engaged in proceedings that would ordinarily come before the judge, or if the ward becomes involved in adversary proceedings in the court on which the judge serves, or a court under its appellate jurisdiction. CJC Rule 3.8(B). Here, because the position involves the judge's father, a close family member, and likely will not interfere with the judge's judicial duties, it would be proper for the judge to undertake the guardianship. Answer choice A is not correct, as a judge who properly becomes a guardian may receive compensation for such service. Answer choice C is not correct, as the fact that the judge will have to appear in court is not enough to prohibit acceptance of the guardianship. Only if the guardian will have to appear in proceedings that would ordinarily come before the judge or before a court on which the judge serves would it be improper. Here, the judge is on the criminal court, so this proceeding would not ordinarily come before the judge. Answer choice D is not correct, as it is permissible for a judge acting as a guardian for a close family member to prepare and sign pleadings, motions, and other papers.

After a high-profile defendant was acquitted of murder, the prosecutor met with the media on the courthouse steps. When asked whether he was satisfied with the verdict, the prosecutor responded: "No; he got away with murder." Is the attorney subject to discipline with regard to addressing the media? Answers: A. No, because he was responding to a media inquiry. B. No, because the trial was over at the time of the statement. C. Yes, because the prosecutor made an extrajudicial statement that the lawyer knows or reasonably should know will be disseminated by means of public communication. D. Yes, because the prosecutor made an extrajudicial statement about the guilt or innocence of the defendant.

Answer choice B is correct. Under MRPC 3.6(a), a lawyer who is participating or has participated in the investigation or litigation of a matter is not permitted to make an extrajudicial statement that the lawyer knows or reasonably should know will be disseminated by means of public communication and will have a substantial likelihood of materially prejudicing an adjudicative proceeding in the matter. However, since the trial was over at the time the prosecutor made this statement, and the prosecution may not appeal the verdict here, the statement can no longer prejudice the proceeding. Answer choice A is incorrect because the fact that the prosecutor was responding to a media inquiry is irrelevant. The relevant fact is that the trial was over. Answer choice C is incorrect because, while such a statement is prohibited during a trial, it was made here after the trial was over. Answer choice D is incorrect because the prosecutor may make such a statement after the conclusion of a trial.

A law firm has 300 lawyers in 10 states. It has placed the supervision of all routine administrative and financial matters in the hands of a non-lawyer administrative assistant. The administrative assistant is paid a regular monthly salary and a year-end bonus of 1% of the law firm's net income from fees. Organizationally, the administrative assistant reports to the managing partner of the law firm. The partner deals with all issues related to the law firm's supervision of the practice of law. Is it proper for the partner to participate in the law firm's use of the administrative assistant's services in this fashion? Answers: A Yes, unless the administrative assistant has access to client files. B Yes, if the administrative assistant does not control the professional judgment of the lawyers in the firm. C No, because the law firm is sharing legal fees with a non-lawyer. D No, because the law firm is assisting a non-lawyer in the unauthorized practice of law.

Answer choice B is correct. Under MRPC 5.4(a), fees may be shared with law firm personnel who are not lawyers through a compensation plan, even if it is based on a profit-sharing arrangement. Under MRPC 5.4(d), a lawyer may not practice in a for-profit firm if a non-lawyer has the right to direct or control the professional judgment of a lawyer. Here, if the administrative assistant does not control the professional judgment of the lawyers in the firm, then the attorney may properly participate in the use of the administrative assistant's services. The fact that the administrative assistant has a compensation agreement based on a profit-sharing arrangement is not improper. Answer choice A is not correct. The administrative assistant would potentially need access to client files to do administrative work. Such access is not improper. Answer choice C is not correct, as MRPC 5.4(a) allows legal fees to be shared with non-lawyers through a compensation plan, even if it is based on a profit-sharing arrangement. Answer choice D is not correct, as the administrative assistant is not practicing law. The administrative assistant is doing administrative work for the firm.

A law firm represented residents who were opposed to the rezoning of a nearby undeveloped parcel of land from rural to retail. A newly hired associate was assigned to work on the case. The associate had previously worked for another law firm, where she had participated in the representation of the landowner in securing the environmental permits necessary for the development of the parcel. The associate, after discussing the matter with the supervisory partner at her new firm, accepted the assignment. Even though the associate believed that accepting the assignment violated the conflict-of-interest rules, the partner concluded that there was not a conflict of interest and that, consequently, the consent of the landowner was not required. When the landowner learned that the associate was working with the residents, he notified the appropriate disciplinary authority. Subsequently, the partner was disciplined for violation of the conflict-of-interest rules. Is the associate also subject to discipline? No, because the associate was acting at the direction of her supervisor. No, because the associate acted in accordance with the partner's resolution of a question of professional duty. Yes, because the associate's participation in the representation of the residents clearly constituted a conflict of interest. Yes, because the associate took actions that she believed violated a professional duty.

Answer choice C is correct. A subordinate attorney is bound by the Model Rules of Professional Conduct even if acting under the direction of a supervising attorney. A subordinate attorney is not in violation of the Model Rules if she acts in accordance with a supervising attorney's reasonable resolution of an arguable question of professional duty. However, in this instance, the associate's participation in the representation of the residents constituted a clear conflict of interest with her former client, the landowner. The associate had formerly represented the landowner in a substantially related matter, and the residents' interests are materially adverse to the landowner's interest. Without the landowner's informed consent, confirmed in writing, there is a clear conflict of interest in the associate's participation in the representation of the residents. Consequently, the associate cannot rely on the partner's resolution of this issue to shield herself from discipline for violation of the conflict-of-interest rules. Answer choice A is incorrect, because a subordinate attorney may be subject to discipline even if she is acting at the direction of a supervising attorney when it is clear that a course of action to be undertaken by the subordinate attorney violates the Model Rules of Professional Conduct. Answer choice B is incorrect because a subordinate attorney is protected from discipline for violation of a Model Rule when there is an arguable question as to the subordinate attorney's professional duty and when the supervising attorney's resolution of the question is reasonable. Here, neither of those conditions is met. Answer choice D is incorrect because a subordinate attorney is not subject to discipline for violation of the Model Rules if she acts in accordance with the supervising attorney's reasonable resolution of an arguable question of professional duty, even if the subordinate attorney believes that such action violates a professional duty. To determine whether the resolution is reasonable, the associate must undertake her own analysis of the applicable law.

A toy manufacturer was sued by the parent of a child injured by one of its products. As the manufacturer's attorney was preparing to respond to a discovery request from the plaintiff, the attorney found a document that was very damaging to his client's case. Prior to complying with the discovery request and turning over the document, the attorney called his opposing counsel and offered to settle the case. The attorney stated that although he believed his client was very likely to win a summary judgment motion, they would settle the case for a modest amount to save the costs of litigation. In fact, the attorney believed his client had no chance of winning a summary judgment motion and was also likely to lose at trial based on the document he had found. The opposing counsel declined the attorney's offer. The attorney turned over the document, and the case proceeded to trial, where judgment was awarded to the plaintiff. Were the attorney's statements to the opposing counsel proper? No, because the attorney did not believe in the truthfulness of the statement. No, because the attorney owed a duty of candor to the opposing counsel. Yes, because the attorney's statement did not constitute a statement of fact. Yes, because the opposing counsel did not accept the attorney's offer.

Answer choice C is correct. An attorney is not permitted to make a false statement of material fact. Statements that constitute "puffing" (i.e., opinions or judgments not made as a representation of fact) are permissible as part of a negotiation. Subjective statements, such as statements about the relative merits of a case, are not statements of material fact. In this case, the attorney's statements regarding the merits of the case would constitute puffing and are thus permissible. Answer choice A is incorrect because the attorney did not make a false statement of material fact; this statement constituted permissible puffing. Answer choice B is incorrect because although an attorney is prohibited from making false statements of material fact, an attorney does not owe the duty of candor to opposing counsel that she owes to the court. Answer choice D is incorrect because although the opposing counsel did not accept the attorney's offer and hence the plaintiff was not harmed by the attorney's statements, the absence of harm does not prevent an attorney's action from being a violation of the Model Rules of Professional Conduct.

During a conversation among friends in his home, an attorney called into question whether a candidate for the office of attorney general met the statutory requirements. Specifically, the attorney stated that he had heard a rumor that the candidate had not been licensed to practice law in the state for the requisite number of years mandated by statute. When asked whether he had investigated this matter, the attorney replied that he didn't need to check the facts, it had to be true. The candidate had, in fact, been licensed to practice law for the statutorily required time period. Is the attorney subject to discipline? Answers: A. No, because the statement did not concern a current member of the judiciary or a candidate for judicial office. B. No, because the statement was not made in a public forum. C. Yes, because the false statement was made with a reckless disregard for its truth or falsity. D. Yes, because the statement concerned a candidate for public office.

Answer choice C is correct. False statements by a lawyer can unfairly undermine public confidence in the administration of justice. The prohibition on such statements applies to the qualifications or integrity of not only a judge or a candidate for judicial office, but also a public legal officer, such as an attorney general, or a candidate for such office. Consequently, a lawyer must not make a statement that the lawyer knows to be false or with reckless disregard as to its truth or falsity (i.e., a false statement made with actual malice) concerning the qualifications or integrity of a judge, adjudicatory officer, or public legal officer, or of a candidate for election or appointment to judicial or legal office. Because the attorney evidenced a reckless disregard for the truth or falsity of his statement regarding the failure of the attorney general candidate to meet the statutory requirements, the attorney is subject to discipline. Answer choice A is incorrect because, as noted with regard to answer choice C, the prohibition on a false statement made with actual malice about the qualifications or integrity of an individual applies not only to a judge or a candidate for judicial office, but also to a public legal officer, such as an attorney general, or a candidate for such office. Answer choice B is incorrect because the prohibition applies to false statements made with actual malice, regardless of whether the statements are made in private or in public. Answer choice D is incorrect because the prohibition on false statements does not apply to any candidate for a public office, but is limited to candidates for judicial or legal offices, such as judges, attorney generals, prosecuting attorneys, and public defenders.

A home builder employed an attorney to draft a contract of sale for a house the builder had constructed. The builder sold the house to a buyer who assumed a personally significant debt in order to purchase the house. After the sale, a friend of the builder's told the attorney that the builder, in violation of the law, had built the house on a landfill. The attorney contacted the builder and urged the builder to discuss the matter with the buyer. Although the builder admitted to the criminal act, the builder refused. The lawyer took no further action. Subsequently, although the landfill did not represent a health threat, the buyer had to sell the house for less than half of what he had paid for it. Is the attorney subject to discipline for failing to reveal the builder's criminal action to the buyer? Answers: A. Yes, because the client had used the attorney's services in furtherance of a crime. B. Yes, because the crime resulted in substantial injury to the financial interests of the buyer. C. No, because of the duty of confidentiality. D. No, because, at the time that the attorney learned of the crime, it had already occurred.

Answer choice C is correct. Subject to a few exceptions, a lawyer is prohibited from disclosing information relating to the representation of a client, whether that information was obtained from a third party or from the client, and whether it was acquired after the termination of the representation. While there is an exception for the disclosure of confidential information to prevent, mitigate, or rectify substantial financial harm to another based on a client's fraud or crime in furtherance of which the client has used the lawyer's services, the exception allows but does not require disclosure. Accordingly, the attorney is not subject to discipline for keeping the information confidential. Answer choice A is incorrect because, although the client had used the attorney's services in furtherance of a crime, the applicable exception that permits disclosure of confidential information in such circumstances does not require such disclosure. Answer choice B is incorrect because, although the crime resulted in substantial injury to the financial interests of the buyer, disclosure is not required. Answer choice D is incorrect because the exception to the disclosure of confidential information based on a client's fraud or crime encompasses situations in which the attorney learns of the crime after it has occurred. However, the exception does not require the attorney to disclose confidential information, but merely permits it.

An attorney served two four-year terms as the state's governor immediately prior to reopening his law office in the state. The attorney printed and mailed an announcement of his return to private practice to members of the bar, previous clients, and personal friends whom he had never represented. The printed announcement stated that the attorney had reopened his law office, gave his address and telephone number, and added that he had been the state's governor for the past eight years. Is the attorney subject to discipline for the announcement? Answers: A Yes, because the mailing included persons who had not been his clients. B Yes, because his service as governor is unrelated to his ability as a lawyer. C No, because the information in the announcement was true. D No, because all of the information was already in the public domain.

Answer choice C is correct. The communication is an advertisement. MRPC 7.1, which covers lawyer advertising, forbids only false or misleading communications, and this communication was neither false nor misleading. Answer choice A is incorrect because although MRPC 7.3 prohibits lawyers from making in-person, live telephone, or real-time electronic contact to solicit for pecuniary gain, this communication does not fall within the banned behavior. It is an advertisement governed by MRPC 7.1 and 7.2. As a result, the communication is permissible so long as it does not contain any false or misleading statements, which this announcement did not. Answer choice B is incorrect, as MRPC 7.1 forbids only false or misleading communications in lawyer advertising. A truthful statement about the attorney's prior service as governor is not likely to create any unjustified expectations and is therefore not misleading. Answer choice D is also incorrect. Information in the public domain can still be either false or misleading. As such, communication of that information might be prohibited under MRPC 7.1, which forbids false or misleading communications in lawyer advertising.

An attorney experienced several instances when clients failed to pay their fees in a timely manner, but it was too late in the representation to withdraw without prejudicing the clients. To avoid a recurrence of this situation, the attorney has drafted a stipulation of consent to withdraw if fees are not paid according to the fee agreement. She proposes to have all clients sign the stipulation at the outset of the representation. Is it proper for the attorney to use the stipulation to withdraw from representation whenever a client fails to pay fees? Answers: A Yes, because a lawyer may withdraw when the financial burden of continuing the representation would be substantially greater than the parties anticipated at the time of the fee agreement. B Yes, because the clients consented to the withdrawal in the stipulation. C No, because a client's failure to pay fees when due may be insufficient in itself to justify withdrawal. D No, unless clients are provided an opportunity to seek independent legal advice before signing the stipulation.

Answer choice C is correct. While in some cases failure to pay fees may be sufficient to justify a withdrawal under MRPC 1.16, in other cases it will not be sufficient. Under MRPC 1.16(b)(5), a lawyer may withdraw if the client refuses to fulfill an obligation regarding the representation, which can include nonpayment of fees. In such cases, however, the client must first be given reasonable warning and an opportunity to satisfy the obligation. The use of an advance-consent stipulation allowing withdrawal in all circumstances in which fees are not paid immediately would not allow for a proper warning or provide an opportunity for the client to satisfy her obligation as required by the rule. Answer choice A is not correct. While a lawyer may seek to withdraw when the representation will result in an unreasonable financial burden on the lawyer, this stipulation attempts to allow the attorney to withdraw whenever fees are not paid according to the fee agreement. A court may determine that one instance of nonpayment does not necessarily constitute an unreasonable burden. Answer choice B is not correct, as even if a client consents to a withdrawal, a court may nevertheless order a lawyer to continue representation. Answer choice D is not correct, as the requirement for seeking independent legal advice applies to the settlement of malpractice claims, not in this context.

A judge, prior to her appointment to the probate court, was a partner in a law firm. The law firm had an extensive probate practice. At the time of the judge's appointment, the law firm had pending before the court to which the judge was appointed numerous matters in which requests were being made for allowances for attorney's fees. When the judge left the law firm, she was paid a cash settlement. She has no further financial interest in any matter handled by the law firm. The judge is now being asked to rule on these requests for allowances for attorney's fees. Is it proper for the judge to rule on these requests? Answers: A Yes, because the judge has no financial interest in the outcome of these cases. B Yes, if these requests are not contested. C No, unless the judge notes on the record in each case her prior association with the law firm. D No, because the judge was associated with the law firm when these matters were pending.

Answer choice D is correct. A judge must disqualify herself if she was associated with a lawyer who participated substantially as a lawyer in the matter during such association. CJC Rule 2.11(A)(6). Answer choice A is not correct because even if the judge has no financial interest in the outcome of the cases, the Code of Judicial Conduct requires recusal when the judge was associated with a lawyer who participated substantially as a lawyer in the matter during such association. Answer choice B is not correct, as the Code of Judicial Conduct requires recusal even if the requests are not contested. Similarly, answer choice C is not correct, as recusal is required and cannot be avoided merely by noting the judge's prior association with the law firm for the record.

A client, during a conference with her attorney in his office about the client's pending divorce, threatened to kill her husband, from whom she was separated. Based on his knowledge of the client, the attorney believed his client's threat to be credible. The attorney took no action to warn the client's husband. Shortly thereafter, the client shot her husband, seriously wounding him. Is the attorney subject to discipline for his failure to warn his client's husband? Answers: A Yes, because the attorney had a duty to warn his client's husband. B Yes, because the attorney's failure to warn the client's husband was the cause of the husband's harm. C No, because, since the client's husband did not die, the client did not accomplish her threat. D No, because the attorney owed his client a duty of confidentiality.

Answer choice D is correct. A lawyer has an ethical duty of confidentiality to his client, which includes protecting confidential communications received from a client. MRPC 1.6(a). Since the communication regarding the client's intention to kill her husband was received by the attorney in the course of his representation of the client in a divorce, the attorney had a duty not to reveal the client's threat, even to the subject of the threat, the client's husband. While the Model Rules allow a lawyer to reveal confidential information concerning the representation of a client to the extent that the lawyer reasonably believes disclosure is necessary to prevent reasonably certain death or bodily harm, the Model Rules do not require disclosure. Accordingly, the attorney here would not be subject to discipline for failing to warn the client's husband. Answer choice A is incorrect because, while a lawyer may reveal confidential information received from a client concerning the representation to the extent that the lawyer reasonably believes disclosure is necessary to prevent reasonably certain death or bodily harm, a lawyer is not obligated to do so. MRPC 1.6(b)(1). Answer choice B is incorrect because, while the causal connection between the attorney's failure to warn the client's husband and the husband's harm would be relevant in a civil action that seeks to impose liability on the attorney, it is not relevant in a disciplinary action based on that failure. Answer choice C is incorrect because whether the client succeeded in accomplishing her threat is irrelevant to the issue of whether the attorney had a duty to warn the client's husband of that threat.

A new attorney was retained by a client to incorporate the client's business, which previously had been operated as a sole proprietorship. The new attorney noticed in the client's file copies of some correspondence from the client to a previous attorney concerning the possibility of that previous attorney incorporating the client's business. The new attorney questioned the client to make certain that any attorney-client relationship between the previous attorney and the client had been terminated. The client told the new attorney, "It certainly has been terminated. When I discussed the matter with my previous attorney six months ago, he asked for a retainer of $1,000, which I paid him. He did absolutely nothing after he got the money, even though I called him weekly, and finally, last week when I again complained, he returned the retainer. But don't say anything about it because my previous attorney is an old friend of my family." Is the new attorney subject to discipline if she does not report her knowledge of the previous attorney's conduct to the appropriate authority? Answers: A. Yes, if the new attorney believes the previous attorney clearly was guilty of professional misconduct. B. Yes, unless the new attorney believes the previous attorney does not usually neglect matters entrusted to him. C. No, if the client was satisfied by the previous attorney's return of the retainer. D. No, unless the client agrees that the new attorney may report the information.

Answer choice D is correct. Generally, a lawyer must report misconduct by another lawyer to the appropriate professional authority when the lawyer has actual knowledge of the misconduct. However, if the lawyer learns of misconduct through a communication protected by the duty of confidentiality, the lawyer is not required to report the misconduct. MRPC 8.3(c). Here, the client has specifically asked the new attorney to keep the information confidential, and she would be subject to discipline for violating the duty of confidentiality if she did report the misconduct. If the client consents to the reporting, however, the new attorney could properly report the misconduct. Answer choice A is not correct, as MRPC 8.3(c) prohibits the disclosure as a violation of the duty of confidentiality. Answer choice B is not correct, as it does not matter whether the previous attorney is or is not usually responsible as an attorney. MRPC 8.3(c) prohibits the disclosure without the client's consent. Answer choice C is not correct because it is irrelevant whether the client was satisfied by the previous attorney's return of the retainer. The new attorney would generally be required to report the misconduct, but she must not in this instance because it would violate her duty of confidentiality to the client.

An attorney was convinced that his client was suffering from dementia. The attorney spoke to his client's family physician and the client's only daughter to determine whether a guardian should be appointed to monitor the client's finances. These were the only discussions the attorney had ever had with either the physician or the daughter. In these discussions, the attorney revealed confidential information about a bank account maintained by the client before learning that the daughter and her mother were estranged because the daughter had stolen from her mother in the past. Was the attorney's revelation of the confidential information proper? Yes, because the attorney was trying to determine whether his client needed a guardian. Yes, because the daughter had relevant information to help determine whether the client needed a guardian. No, because the attorney should not have disclosed confidential information about a client to others without prior court approval. No, because the attorney did not first determine whether either the doctor or his client's daughter might act adversely to his client's interests.

Answer choice D is correct. Information that relates to the representation of a client with diminished capacity is protected by the duty of confidentiality. Thus, an attorney may not generally disclose such information without authorization. When taking protective action such as seeking the appointment of a guardian, however, an attorney is impliedly authorized to reveal information about the client, but only to the extent reasonably necessary to protect the client's interests. The attorney should determine whether it is likely that the person consulted will act adversely to the client's interests before discussing matters related to the client. In this case, the attorney did not act properly because he did not make such a determination before revealing confidential information. Answer choice A is incorrect because, while the attorney may disclose confidential information when taking protective action for a client with diminished capacity, he should first consider whether it is likely that the person consulted will act adversely to the client's interests. Here, the daughter, based on her past interaction with her mother, could take action that would be antithetical to her mother's financial interests. Answer choice B is incorrect because, although the daughter likely had relevant information, whether the information is relevant is not the only consideration. Answer choice C is incorrect because prior court approval is not required.

The judicial district in which a judge sits has a rule that allows litigants two postponements as a matter of right. After that, a litigant who moves for a postponement must convince the presiding judge that a postponement is appropriate. The judge routinely grants additional postponements because, in her view, "What harm is done if one of the litigants wants a postponement? The worst that can happen is that the parties have more time to negotiate and thus are more likely to settle." Are the judge's actions proper? Answers: A Yes, because the judge is exercising her judicial discretion. B Yes, because a party objecting to a postponement can seek appellate review. C No, because judges have no official obligation to encourage private settlements. D No, because the judge should expedite the determination of matters before her.

Answer choice D is correct. Under CJC Rule 2.5(A), a judge is required to perform judicial and administrative duties diligently, which includes monitoring and supervising cases in ways that reduce or eliminate dilatory practices, avoidable delays, and unnecessary costs. CJC Rule 2.5, cmt 4. A blanket policy granting additional postponements conflicts with the judge's duty to expedite the determination of matters before her. Answer choice A is not correct because the judge's discretion is circumscribed by a duty to expedite the determination of matters before her. Answer choice B is not correct, as it is irrelevant to the determination of whether the judge's actions are proper under the Code of Judicial Conduct that a party can seek appellate review. What controls is the judge's ethical duty as a judge to expedite the determination of matters before her. Answer choice C is not correct, as it is not improper for a judge to attempt to encourage private settlements. The difficulty here is that the judge has imposed a blanket policy of granting additional postponements that may lead to avoidable delays and unnecessary costs, which conflicts with the judge's duty under CJC Rule 2.5(A).

In representing a client in litigation involving a boundary dispute, an attorney, after consultation with and approval by the client, employed a surveyor. The attorney, who had used and compensated the surveyor in previous, similar situations for other clients, described the purpose of the survey and the party she represented to the surveyor. The retainer agreement between the attorney and the client specified that the client was responsible for payment of all litigation expenses. The surveyor performed a survey of the disputed boundary and submitted an invoice to the attorney for the agreed-upon amount. Prior to payment of this invoice, the client, in direct conversation with his neighbor, reached an agreement over the boundary between their properties. The client paid the attorney her fee as agreed upon in the retainer agreement but refused to pay the attorney for the cost of the survey. Is the attorney likely subject to civil liability to the surveyor for the unpaid invoice? No, because the client was responsible for the payment of all litigation expenses. No, because the client was consulted about and approved the hiring of the surveyor. Yes, because an attorney is primarily responsible for litigation expenses. Yes, because of the nature of the services rendered by the surveyor.

Answer choice D is correct. While generally an agent who enters into a contact with a third party on behalf of a disclosed principal is not liable on the contract, an attorney who acts on behalf of a client can be liable for the payment of goods and services typically used by lawyers, such as a stenographer, appraiser, or surveyor, because such persons are likely to reasonably rely on the attorney's, rather than the client's, credit. Answer choice A is incorrect because, although the client was contractually obligated to pay litigation expenses (as is generally required by the Model Rules of Professional Conduct except in the case of a contingency fee arrangement or an indigent client), the attorney is liable to the surveyor; note, though, that the attorney is entitled to seek reimbursement from the client for the payment of the surveyor's bill. Answer choice B is incorrect because, although the client was consulted about and approved the hiring of the surveyor, the attorney is liable to the surveyor, even though the attorney is entitled to seek reimbursement from the client for payment of the surveyor's bill. Answer choice C is incorrect because the Model Rules of Professional Conduct generally require the client, not the attorney, to bear responsibility for the payment of litigation expenses.

An attorney is a voting member of the legislation committee of a consumer-based law reform group that drafts and advocates the passage of proposed statutes on food safety. The law reform group is currently debating a draft statute that sets quality and safety standards for growth hormones administered to chickens, turkeys, and other poultry. The attorney is also engaged in the private practice of patent law. She regularly represents a biotechnology firm. Using the techniques of genetic engineering, the biotechnology firm invents, develops, and sells a variety of patented growth hormones. The attorney herself has obtained patents on some of these hormones for the biotechnology firm. If enacted into law, the law reform group's proposed statute on poultry hormones could materially increase the biotechnology firm's hormone sales because it is the only firm whose hormones would meet the statute's quality and safety requirements. Would it be proper for the attorney, as a member of the law reform group's legislation committee, to participate in the debate on, and to cast her vote on, the proposed statute? Press Enter or Space to submit the answer A No, because the statute could materially benefit the biotechnology firm. B No, because the attorney may not serve as a member of the law reform group while representing the biotechnology firm. C Yes, provided that she informs the legislation committee that she represents an unnamed client whose interests could be materially benefited by the statute. D Yes, provided that she informs the legislation committee that she represents the biotechnology firm, whose interests could be materially benefited by the statute.

It would be proper for the attorney to participate in the debate and cast her vote on the proposed legislation, provided that she informs the committee that she represents a client whose interests could be materially benefited by the statute. A lawyer may participate in a law reform activity that will affect the interests of the lawyer's client. [ABA Model Rule 6.4] When a lawyer knows that a client will be materially benefited by the activity, the lawyer must disclose that fact, but she need not name the client. (A) is wrong because a lawyer is not prohibited from engaging in a law reform activity that might benefit her client. (B) is wrong because a lawyer is not prohibited from participating in a law reform activity, unless the participation would create an impermissible conflict of interest. [ABA Model Rule 1.7(a)] That is not the case here. A client who hires a lawyer does not thereby purchase the right to control the lawyer's views and activities in all contexts. [See ABA Model Rule 1.2(b)] The attorney may even advocate new legislation that she thinks is sound that would harm the biotechnology firm's sales. [See ABA Model Rule 6.4] (D) is wrong because the attorney need not disclose the name of her client; simply disclosing the fact of representation will inform the legislation committee of her possible bias.

A client hired a lawyer to draft a will for him. The client willed his entire estate to a 43-year-old widow. The client told the lawyer in confidence that he was neither a relative nor a friend of the widow. The client explained that he felt a moral obligation to the widow because he had killed her husband, and he had never become a suspect or confessed his sin to anyone. One day after signing the will, the client committed suicide. In due course, all of the client's assets were distributed to the widow, and the probate court closed his estate and discharged his executor. The lawyer never told the widow or anyone else that the client had confessed to killing the widow's husband. Now, a few years later, an enthusiastic young prosecutor is charging an innocent man with murdering the widow's husband in the first degree with aggravating circumstances, and the prosecutor is seeking the death penalty. May the lawyer voluntarily tell the innocent man's defense counsel what his client told him in confidence about killing the widow's husband? Press Enter or Space to submit the answer A Yes, the lawyer not only may, but he must, tell the defense counsel what the client told him. B Yes, the lawyer may tell, but he would not be subject to discipline if he decides not to do so. C No, the lawyer would be subject to discipline if he told defense counsel because the attorney-client privilege survives the death of the client. D No, because the client's confidential confession to the lawyer would be inadmissible hearsay if offered against the prosecution in the murder trial.

The controlling doctrine in this case is the lawyer's ethical duty of confidentiality, not the attorney-client privilege. The lawyer needs to know whether he can voluntarily reveal the client's confession, not whether he would be forced to do so if he were put on the witness stand in a court. ABA Model Rule 1.6(b)(1) states the applicable exception to the ethical duty of confidentiality: A lawyer may reveal confidential information if the lawyer reasonably believes that doing so is necessary to prevent reasonably certain death or substantial bodily harm. One might quibble whether the innocent man's death is "reasonably certain" when his trial has not even started, but surely the ethics rule should not be read to require the innocent man to order his last meal before being loosed from the executioner's grip. (A) is wrong because ABA Model Rule 1.6(b)(1) gives the lawyer discretion to reveal the client's confession; the Rule does not force him to do so. [See comment 15 to ABA Model Rule 1.6] (A few states go farther and require disclosure to prevent death or substantial bodily harm, but they are a small minority.) (C) is wrong for two reasons. First, the applicable doctrine is the ethical duty of confidentiality, not the attorney-client privilege. Second, even if the privilege were the applicable doctrine, who could claim it in this situation? The client cannot because he is dead. The client's executor cannot because the client's estate was closed and the executor was discharged. The lawyer cannot claim it because a lawyer's right to claim the privilege is only derivative from the client. (D) is wrong for two reasons. First, the admissibility of this hearsay is irrelevant to the ethics issue. Second, the client's confession would likely be admissible if offered by the innocent man against the prosecution because it is a declaration against penal interest by an unavailable declarant, and the client's will and suicide are independent evidence of the confession's trustworthiness. [See Fed. R. Evid. 804(b)(3); see also Chambers v. Mississippi, 410 U.S. 284 (1973)-due process violation where another man's confession was excluded in a murder trial]

A young associate was assisting a senior partner in writing the reply brief in an appeal for one of the partner's clients. In doing the legal research, the associate discovered a recent case from the controlling jurisdiction that had not been cited in the adversary's brief. In the associate's opinion, the case was directly opposed to the position of the partner's client. The associate asked the partner about citing it in the reply brief, but the partner explained that, in his view, the case was not directly on point and did not have to be cited. The associate and the partner argued back and forth at some length and finally decided to submit the question to one of the other senior partners in the firm for a fresh view. That partner sided with the other partner, and the reply brief was filed without mentioning the case. May the associate write a short letter to the appellate court and the adversary lawyer, explaining his position and enclosing a copy of the case? Press Enter or Space to submit the answer A Yes, because the associate had a duty to call the case to the court's attention. B Yes, because the associate must not allow another person to interfere with his professional judgment. C No, because the associate must not communicate with a court ex parte about the merits of a pending case. D No, because the associate should abide by the partner's resolution of the matter.

The associate should abide by the partner's resolution of the matter. A subordinate lawyer does not violate the Rules of Professional Conduct by acting in accordance with a supervisor's reasonable resolution of an arguable question of professional duty. [ABA Model Rule 5.2(b)] Here, it seems clear that the question was arguable because the third attorney called in to determine the relevance of the case also felt it was not on point. (A) is wrong because the associate only has a duty to call the case to the court's attention if the case is directly on point. That is a debatable question, and the associate's supervisors have determined the case is not directly on point. Thus, the associate need not reveal the case. (B) is wrong because this is not the situation intended to be addressed by the rule against allowing a third party to influence the lawyer's judgment, which usually arises when a third party pays the lawyer's fees to represent another. Of course, a subordinate lawyer should be influenced by his supervisor. That is not an excuse for clearly unethical conduct, but on a debatable issue, such as the one presented here, the subordinate lawyer is free to defer to the supervisor's judgment. (C) is wrong because in most jurisdictions a lawyer may communicate in writing with the court about the merits of a pending case if he sends a copy to opposing counsel. This communication is not considered ex parte. [See Restatement §113, comment c]

An attorney worked at the United States Department of Labor and was responsible for compiling certain corporate safety records into an annual report containing the accident statistics. The report is used internally and in discussions with companies, but it is not distributed to the general public. However, a person may obtain a copy of the report by filing a formal request under the Freedom of Information Act. During the last three years, Company A has had more accidents than any of the other reporting companies. Six months ago, the attorney left the Labor Department and took a job with a private law firm. Recently, a person came to the attorney seeking representation in a suit against Company A for injuries he sustained while working at Company A's factory. The attorney agreed to represent the client. Is the attorney subject to discipline? Press Enter or Space to submit the answer A Yes, because he obtained relevant information about Company A while working as a government attorney. B Yes, because the attorney did not obtain the consent of the Department of Labor. C No, because the information is available by formal request under the Freedom of Information Act. D No, if the attorney does not use the information obtained while employed as a government attorney to the material disadvantage of Company A.

The attorney is not subject to discipline for taking the case because the relevant information he obtained while working as a government attorney is not confidential. The general rule is that a government lawyer who receives confidential government information about a person must not later represent a private client whose interests are adverse to that person, if the information could be used to the material disadvantage of that person. [ABA Model Rule 1.11(c)] The rule covers only "confidential" information, which means information that the government is prohibited from revealing or has a privilege not to reveal, and which is not otherwise available to the public. Here, because the information is available under the Freedom of Information Act, it is not confidential. In fact, any attorney representing the client could obtain the information; thus, the attorney is free to use it. (A) is wrong because an attorney is not barred from ever working on a case where he gained any relevant information while working for the government. To bar representation, the information must be confidential. (B) is wrong because this type of consent is required when the attorney takes on a representation in private practice in a matter in which the lawyer participated personally and substantially while in government service. A "matter" is a set of specific facts involving specific parties. Here, the attorney was not involved in any matter while in government service that concerned the client's claim against the chemical company. (D) is wrong because the information is not confidential and thus can be used against the chemical company. Furthermore, even if the information were confidential, mere nonuse would not be sufficient; the attorney would not be permitted to represent the client.

An attorney was appointed by the court to defend a client at his criminal trial for second degree murder. The attorney started interviewing potential witnesses. When she interviewed the client's landlord, the landlord said that on the night of the murder, the client came home very late and was wearing a shirt covered with blood. The landlord died before trial without speaking to state authorities. Which of the following best states what the attorney should do with respect to the information she has learned from the landlord? Press Enter or Space to submit the answer A The attorney should voluntarily reveal the information to the prosecutor prior to trial because the death of the landlord has made it impossible for the prosecutor to obtain the information in any other way. B The attorney should urge the client to allow her to reveal the information to the prosecutor, and if the client refuses, the attorney should withdraw. C The attorney should keep the information in confidence unless the client authorizes her to reveal it, even though the death of the landlord has made it impossible for the prosecutor to obtain the information in any way other than from the attorney. D The attorney should use her own best judgment about how to treat the information; it is neither privileged nor confidential because it was not given to her by her client or by an agent of her client.

The attorney should keep the information in confidence unless the client authorizes her to reveal it, even though the death of the landlord has made it impossible for the prosecutor to obtain the information other than from the attorney. The attorney obtained this information from the landlord in the course of representing her client; therefore, it is subject to the attorney's duty of confidentiality. Absent the consent of the client, an attorney must not reveal any information relating to the representation of the client. [ABA Model Rule 1.6] (A) is incorrect because an attorney has no ethical obligation to reveal harmful facts, and, in fact, may be disciplined for doing so. (B) is incorrect because, as noted above, there is no duty to reveal this information; thus, there is no obligation to urge the client to reveal the information or withdraw. (D) is incorrect because this information is confidential. The ethical duty of confidentiality covers more kinds of information than the attorney-client privilege, which covers only confidential communications between the attorney and client. The ethical duty of confidentiality covers any information the attorney obtains relating to the representation of the client, no matter what the source of the information.

For many years, a tax attorney has handled all of the tax work for his client, a sculptor. One evening, the sculptor invited the attorney to his studio to discuss some tax returns that had to be filed the next day. In the studio, the attorney saw a small sculpture that would be perfect for his office. At the close of their tax discussion, the attorney offered to buy the sculpture for $10,000, its approximate fair market value. The sculptor told the attorney that it was not for sale. In due course, the attorney sent the sculptor a bill for a $750 fee for the tax work. A few days later, the small sculpture was delivered to the attorney's office with a note from the sculptor, saying that he hoped the sculpture would satisfy the recent bill, and he wanted the attorney to have the sculpture as a token of his gratitude for the excellent tax advice. Would the attorney be subject to discipline for accepting the small sculpture from the sculptor? Press Enter or Space to submit the answer A Yes, because the gift is of significant monetary value. B Yes, because the value of the sculpture is far out of proportion to the $750 worth of work the attorney did for the sculptor. C No, because the attorney did not solicit the gift. D No, because the $10,000 is only an approximation of market value.

The attorney would not be subject to discipline for accepting the sculpture because he did not solicit the gift. Although ABA Model Rule 1.8(c) prohibits a lawyer from soliciting a substantial gift from a client when the lawyer is not related to the client, it does not prohibit a lawyer from accepting an unsolicited gift from a client, even if the gift is substantial (although the gift may be voidable for undue influence). Moreover, comment 6 to ABA Model Rule 1.8 states that a lawyer may accept a gift from a client if the transaction meets general standards of fairness. Here, the attorney did not solicit the gift, and there are no facts to suggest undue influence or unfairness. Thus, the gift is proper. (A) is wrong because it is too broad. A lawyer may accept a gift of substantial value from a client if the conditions stated above are satisfied. (B) and (D) are wrong because the value of the attorney's recent work and the value of the sculpture are irrelevant. The attorney did not charge more than the $750. In addition to discharging the $750 fee bill, the sculptor obviously intended to make a gift to the attorney in gratitude for years of work in the past.

An attorney has just opened an office in a town where he does not know many people and has few contacts. The attorney has just heard that a group of townspeople, including a former client, have been arrested and are being held at the county jail for conducting a noisy demonstration outside the local high school to protest an impending strike by the teachers. The attorney strongly believes that the arrests were politically motivated and that the demonstrators have been deprived of their freedom of expression in violation of the First Amendment. He also realizes that he might gain some favorable public exposure if he were to help the arrestees. He therefore goes down to the county jail and offers his legal services, free of charge, to any of the arrested demonstrators who want legal assistance. Are the attorney's actions proper? Press Enter or Space to submit the answer A Yes, because one of the arrested demonstrators was a former client. B Yes, because the attorney was offering his services free of charge. C No, because to do so would involve in-person solicitation of legal business. D No, because he was motivated at least in part by a desire to obtain publicity for his law practice.

The attorney's actions are proper because he offered his services free of charge. Generally, a lawyer is prohibited from seeking employment by initiating live person-to-person contact with a person known to need legal services in a particular matter. However, this prohibition applies only when "a significant motive" for the solicitation is the lawyer's pecuniary gain. [ABA Model Rule 7.3(b)] Thus, a lawyer who volunteers to represent someone free of charge is not subject to discipline for solicitation. (A) is incorrect because, as discussed above, it is not necessary for the attorney to have previously represented any of the demonstrators in order for his actions to have been proper. (C) is incorrect because, as discussed above, this situation falls within an exception to the ban on live person-to-person solicitation. (D) is incorrect because the attorney's actions are proper even if he is motivated by a desire to obtain publicity, provided that this is not a substantial motive for his offer.

In an effort to prevent homosexual persons from moving to their small town, a city council passed a zoning ordinance prohibiting the use of any dwelling house as a residence for two or more adults of the same sex unless they are related to each other. Violation of the ordinance is a misdemeanor and carries a fine of $10,000. A landlord who owns several rental houses in the city and does not want his potential renter pool limited hired an attorney to advise him. The attorney advised the landlord that the ordinance could probably be overturned under the state constitution, but that one would have to obtain legal standing to challenge the ordinance. The attorney advised that one way to obtain legal standing would be for a landlord to bring an appropriate action for declaratory judgment, and another way would be to simply violate the ordinance and raise the constitutional challenge as a defense to its enforcement. After receiving this advice, the landlord promptly rented one of his houses to two gay men. The landlord was cited for violation of the ordinance. The landlord subsequently brought an action to challenge the ordinance, and the ordinance was held unconstitutional. Was the attorney's conduct proper? A Yes, because violating the ordinance was one of the few ways to gain legal standing to challenge the constitutionality of the law. B Yes, because the ordinance was ultimately held unconstitutional. C No, because the attorney counseled and assisted the landlord in conduct the attorney knew was illegal. D No, because the attorney did not advise against renting houses to unrelated adults of the same sex.

The attorney's conduct was proper because violating the ordinance was one of the ways to gain legal standing to challenge the constitutionality of the ordinance. A lawyer must not counsel or assist a client in conduct that the lawyer knows is criminal or fraudulent. However, a lawyer may counsel or assist a client to make a good faith effort to determine the validity, scope, meaning, or application of a law even if it requires the client to disobey the law. [ABA Model Rule 1.2(d)] This situation usually arises when a lawyer is asked how a particular law may be challenged, and the lawyer advises the client on ways to obtain legal standing, which include disobedience of the law. Here, the attorney was merely advising the landlord on methods of obtaining legal standing, including renting houses in violation of the ordinance. (B) is wrong because the ultimate outcome of the challenge is irrelevant. It is important that the attorney believed there was a good faith basis for challenging the validity of the ordinance, but whether the challenge succeeds does not determine the propriety of his conduct. (C) is wrong because, as discussed above, there is an exception to this rule for a good faith effort to determine the validity of a law. (D) is wrong because there is no affirmative duty to counsel the client in this way. In fact, as discussed above, there is an exception for a good faith challenge to the law that would permit the attorney to do just the opposite.

A swimming coach was charged with assault of another coach. The swimming coach hired a criminal attorney to defend him. Subsequently, the swimming coach pleaded not guilty and was released on his own recognizance. At his first trial, a jury was empanelled, and the prosecutor was almost finished presenting the testimony of her first witness when a signal from her electronic pager interrupted her. The trial judge granted her request for a short recess, at the end of which the prosecutor told the judge that her office had instructed her not to proceed with this case at this time. The judge responded that if the prosecutor stopped now, the defendant would go free. When the prosecutor indicated that she understood, the judge entered a judgment of acquittal and set the swimming coach free. Twenty days later, the prosecutor recharged the swimming coach with the same offense. The swimming coach hired his original criminal attorney to defend him. The same judge presided over the second trial. The swimming coach's attorney made no pretrial motions. This time the prosecutor did not falter, and in due course the jury at the second trial found the swimming coach guilty as charged. The judge sentenced him to prison for the period required by law, but she stayed the sentence and released him on his own recognizance pending appeal. The swimming coach reluctantly paid the criminal attorney's bill for the second trial-$5,000. However, the swimming coach hired a new lawyer for the appeal, and in due course the appellate court reversed the conviction and set aside the prison sentence. The appellate court's opinion stated it had never seen a clearer double jeopardy violation. Will the swimming coach's original criminal attorney be subject to civil liability in a legal malpractice action brought by the swimming coach for having missed the double jeopardy issue? Press Enter or Space to submit the answer A No, because the swimming coach never served jail time as a result of the original attorney's error. B No, even if the swimming coach proves by a preponderance of evidence that he did not commit the assault on the opposing coach. C Yes, provided that the swimming coach proves by a preponderance of evidence that he did not commit the assault on the opposing coach. D Yes, but the swimming coach can recover only nominal damages.

The criminal attorney will be subject to civil liability in a legal malpractice action brought by the swimming coach for failing to object to the second trial on double jeopardy grounds. A reasonably competent criminal defense attorney would know that a defendant is put in jeopardy when a jury is empanelled and sworn, not to mention that the prosecutor started presenting her case-in-chief. The swimming coach was obviously a proper plaintiff, and he was injured by the criminal attorney's error; he should be able to recover at least part of the $5,000 attorneys' fee, plus damages for his anguish and for the reputational injury caused by the conviction at the second trial. Note that (C) provides that in the malpractice action the swimming coach must prove by a preponderance of the evidence that he was innocent of the underlying criminal offense. That is required by the law of most states that have ruled on the issue. [See Restatement §53, comment d] Observe that in this particular case, a good argument can be made for allowing the swimming coach to recover even without proof of innocence. Here, the malpractice was the criminal attorney's failure to object to the second trial. If the attorney had acted competently, the second trial would never have taken place, and the swimming coach would have lawfully gone free, even if he were unquestionably guilty of the assault. [See Levine v. Kling, 123 F.3d 580 (7th Cir. 1997)-Judge Posner's dictum] (A) and (D) are incorrect because they overlook the $5,000 fee and other less tangible injuries the swimming coach suffered. (B) is incorrect for the reasons stated above with respect to (C).

Continuously since 1910, the law firm of Alpha & Beta has practiced under that name. The founders of the firm are long dead. No partner named Beta now practices with the firm. Two partners named Alpha were practicing with the firm, but one recently left because she was appointed to the state supreme court. May the firm continue to use the name Alpha & Beta? Press Enter or Space to submit the answer A No, because no partner named Beta now practices with the firm. B No, because one partner named Alpha left the firm to enter public service. C Yes, unless the firm name would be misleading. D Yes, even if the firm name will mislead some prospective clients.

The firm may continue to use the name Alpha & Beta if it is not misleading. A firm may practice under a trade name, provided that the trade name is not misleading. [Comment 5 to ABA Model Rule 7.1] (A) is wrong because a firm may continue using the name of a deceased partner. [Id.] (B) is wrong because the person who left was not a name partner. Generally, when a name partner enters public service and is not in private practice for a substantial period, the firm must cease using that person's name. [Comment 8 to ABA Model Rule 7.1] Here, the Alpha surname refers to the deceased founder and not the partner who was appointed to the state supreme court. If, however, the use of the Alpha name in the firm name would mislead potential clients (e.g., by making them think they could gain an advantage in the state supreme court by hiring that firm), then continued use of the name would violate the ethics rules. [See ABA Model Rule 7.1] (D) is wrong because the rules on firm names are subject to the more general provisions on misleading communications.

A judge sits on a federal appellate court. He and two other federal judges heard a diversity of citizenship case in which they were required to interpret a state statute concerning the marital communications privilege. The judge's two colleagues wrote the majority opinion, in which they concluded that the statute gives only the witness-spouse the right to claim the privilege. The judge wrote a vigorous and scholarly dissent, arguing that the statute gives both spouses the right to claim the privilege. Later, a state senator introduced a bill to amend the statute to reflect the judge's position. The state senate invited the judge to testify about the public policy reasons for giving both spouses the right to claim the privilege. May the judge testify? Press Enter or Space to submit the answer A Yes, but only if the two judges who wrote the majority opinion are also allowed to testify. B Yes, because a judge may engage in activities designed to improve the law. C No, because a judge must not become involved in politics, subject to certain exceptions that do not apply here. D No, because a judge is not allowed to make public statements about disputed propositions of law, except when acting in his judicial capacity.

The judge may testify at a public hearing in connection with matters concerning the law. [CJC Rule 3.2(A)] (A) is wrong because there is no rule requiring "equal time." (C) is wrong because it is overbroad. The general rule against judicial involvement in politics limits only some types of political activities, not including legislative testimony. [CJC Canon 4] (D) is wrong because, with respect to issues that are likely to come before the court, a judge is prohibited from making pledges, promises, or commitments that are inconsistent with the impartial performance of his duties. [CJC Rule 4.1(A)(13)] That Rule does not apply here because the judge's testimony, which would be designed to improve the law, would not constitute a promise that is inconsistent with the performance of his adjudicative duties.

An attorney is admitted to practice only in State A, where he specializes in securities and real estate finance law. In that role, the attorney advised his client that the law of State B did not require the client to include information about certain mineral rights in a disclosure statement that the client had to file in State B in order to sell some real estate limited partnership interests to State B citizens. Acting on the attorney's advice, the client did not disclose the information and did sell partnership interests to State B citizens. Later, the attorney became a full-time trial court judge in State A. Later still, State B brought a criminal action against the client for failing to disclose the mineral rights information in his State B disclosure statement. One of the client's defenses is that he lacked the necessary criminal intent because he was acting in good faith based on the advice of his counsel, the attorney who is now a judge. The client needs the judge's testimony to prove that the judge did indeed advise him that he was not required to disclose the mineral rights information. The judge, in State A, is beyond the subpoena power of the State B court. May the judge voluntarily testify on behalf of the client? Press Enter or Space to submit the answer A No, because judges are disqualified from serving as witnesses in criminal cases. B No, because he is not admitted to practice in State B, and his testimony about State B law would be inadmissible. C Yes, because a judge may testify as a witness, except in his own court or one under its appellate jurisdiction. D Yes, because his testimony would concern the giving of the advice, not his client's character.

The judge may testify because he is testifying to facts, not the defendant's character. CJC Rule 3.3 prohibits a judge from testifying voluntarily as a character witness, but it says nothing about serving as an ordinary fact witness. (A) is wrong because there is no such rule. Judges are not disqualified from testifying in criminal cases. (B) is wrong because an attorney need not be admitted to practice in State B in order to advise a client about State B law. Even if that were untrue, the judge's testimony would still be admissible as evidence of the client's lack of criminal intent. (C) is wrong because it states a nonexistent rule. While a judge is not competent to be a witness at a trial over which he himself is presiding [see Fed. R. Evid. 605], no rule forbids a judge from serving as an ordinary fact witness in a case that is pending before a different judge in his own court or a court that is under his court's appellate jurisdiction.

A lawyer is interested in obtaining legal business from a mineworkers' union that has many members in the state in which the lawyer practices. As a result of a recent mine fire and explosion in which several union members were killed, the union has succeeded in persuading the appropriate state agency to bring an administrative action against the company that owns the mine for failing to install smoke detectors, which might have saved some lives in the disaster. Although the lawyer is in no way involved in the case, he sees this as an opportunity to obtain future business from the union by showing the union that he is strongly on its side in the mine disaster case. The lawyer telephones a popular call-in radio show, and says that he is shocked and appalled at the callousness of the mining company that caused the recent disaster in which so many miners were killed. He further opines that the mining company was willful and wanton in its failure to install smoke detectors, and expresses hope that the company will not be allowed to escape the consequences of its despicable conduct. Without the lawyer's knowledge or consent, his statement was later printed in several news-papers in the state. Is the lawyer subject to discipline for his conduct? Press Enter or Space to submit the answer A Yes, because he was substantially motivated by his desire to attract fee-paying business. B Yes, because lawyers must not make public comments concerning pending litigation. C No, because he did not make any false or misleading claims about himself or his services. D No, because the statement was printed in the newspapers without his knowledge or consent.

The lawyer is not subject to discipline because he did not make any false or misleading claims about himself or his services. Lawyers, like other citizens, have the right to express their views in the media on newsworthy issues. Even if a lawyer's sole purpose in seeking media publicity is to lure clients, the state may not impose professional discipline on the lawyer absent a compelling state interest. A lawyer who uses the media to lure clients may, however, be disciplined for making statements or claims that are false or misleading about the lawyer or his services. [ABA Model Rule 7.1] Here, the lawyer made no statements about himself or his services other than the fact that he is a lawyer and his opinion about the incident. There is nothing false or misleading in his communication. (A) is wrong because, as discussed above, the fact that the lawyer was motivated by the desire to attract fee-paying business is irrelevant. This is not a case of live person-to-person solicitation, which is the only circumstance in which this consideration is relevant. (B) is wrong because it is overbroad. Lawyers can and do make public statements about pending litigation all the time. Lawyers who are involved in a proceeding cannot make statements that they know will have a substantial likelihood of materially prejudicing an adjudicative proceeding. [ABA Model Rule 3.6(a)] That is not the case here; the lawyer has no reason to believe his opinion will materially prejudice the state agency bringing the action. (D) is wrong because the fact that the statement was printed in the newspapers is of absolutely no consequence; it does not affect the propriety of the lawyer's behavior regardless of whether he gave consent.

The Department of Children's Services ("DCS"), a state agency, has removed a woman's children from her home and is attempting to terminate her parental rights. The woman retains a lawyer to fight DCS's actions. Prior to the first hearing on the matter, a lawyer from DCS contacted the woman's brother to set up an interview. The lawyer told the brother that he wanted to talk to him about his sister, and that the interview would last for about one hour. The brother, who is childless, had never had any contact with DCS before, and did not know what his duties were. He told the lawyer he would get back to him. The brother then called his sister's lawyer. He told her that he does not want to talk to DCS because he is afraid he might inadvertently say something that will hurt his sister's case. He asked his sister's lawyer how he should proceed. She advised him simply to refuse the interview, because he was not required to talk to them and his sister would be better off if he did not. Is the lawyer subject to discipline? Press Enter or Space to submit the answer A Yes, because she is attempting to secure the noncooperation of a witness. B Yes, because she gave advice to an unrepresented party. C Yes, because she did not advise the brother to seek the advice of independent counsel. D No, because the brother is her client's brother and he did not want to hurt her case.

The lawyer is not subject to discipline because of the brother-sister relationship. A lawyer may advise a person not to voluntarily give information to an opponent or other party if the person is a relative of the client and that person's interests will not be harmed by not volunteering the information. [ABA Model Rule 3.4(f)] Here, there is nothing in the facts to suggest the brother's interests would be harmed by not volunteering the information about his sister. (A) is wrong because of the exception stated above. (B) is wrong because this type of advice is not prohibited. If the facts indicated that the brother's interests might be harmed by not granting the interview (e.g., if he had a matter pending with DCS that might be negatively influenced by his noncooperation), the attorney could not have advised him as she did and should then have advised him to seek independent counsel. (C) is wrong for the reason just stated.

A lawyer practices real estate law in an old-fashioned jurisdiction in which almost every real estate transaction requires the services of one or more lawyers. The lawyer is also licensed by the state as a real estate broker. The lawyer conducts her law practice and her real estate brokerage business in a single office, using one secretary and one paralegal as her support staff. The lawyer specializes in small, relatively old apartment buildings that are not in peak condition. They make good investments because they can be bought cheap, fixed up, and leased at favorable rates. When the lawyer hears that an owner of a suitable building is looking to sell, she visits them in person and asks them to consider using her to find a buyer. After an owner signs her up as their real estate broker, the lawyer lets them know that she can also do the necessary legal work-the title search, the financing documents, the land transfer documents, and the like. Is the lawyer subject to discipline? Press Enter or Space to submit the answer A Yes, because a person who is engaged in full-time law practice must not conduct a related business from a single office. B Yes, because a person who offers legal services along with real estate brokerage services must not engage in face-to-face solicitation of persons known to need real estate brokerage services. C No, because the lawyer's real estate brokerage services are ancillary to her law practice, and the two operations are conducted from a single office. D No, so long as her face-to-face pitch to the owners of apartment buildings is truthful and not misleading.

The lawyer's real estate brokerage business is a "law-related service" within the meaning of ABA Model Rule 5.7, and the lawyer offers her real estate brokerage services "in circumstances that are not distinct from" her provision of legal services. [See ABA Model Rule 5.7(a)(1)] That means that she must follow the rules of legal ethics in her real estate brokerage work as well as her law work. [Id.] One of the legal ethics rules forbids a lawyer from initiating live person-to-person contact with a person known to need legal services in a particular matter when a significant motive for doing so is the lawyer's pecuniary gain. [See ABA Model Rule 7.3(b)] Therefore, the lawyer must not initiate face-to-face contact with potential real estate clients to interest them in using her brokerage services. (A) is wrong because it overstates the rule expressed in ABA Model Rule 5.7. (C) is wrong because it turns ABA Model Rule 5.7 on its head-because the lawyer is offering her ancillary service in circumstances that are not distinct from her legal service, she must follow the legal ethics rule for both kinds of service. (D) is wrong because the lawyer's face-to-face pitches violate the no-solicitation rule even if her statements are truthful and not misleading. [Compare ABA Model Rule 7.1 with ABA Model Rule 7.3(b)]

In a trade secret action against a corporation pending in a United States district court, the plaintiff's lawyer gave timely notice that on July 22 at 9 a.m., he would take the deposition of the corporation's vice president for manufacturing and marketing. From earlier discovery in the case, the plaintiff's lawyer had good reason to believe that the vice president's testimony would prove that the corporation had stolen and was using the plaintiff's trade secrets. On the appointed day, the defense lawyer showed up with no witness; she explained that the vice president was a very busy man and had been unavoidably detained on a trip to one of the corporation's factories in Asia. The plaintiff's lawyer rescheduled the deposition for August 3, and the defense lawyer promised to have the witness available that day. On August 3, the defense lawyer again showed up with no witness, explaining that he had to take his aged mother to the doctor that day. The plaintiff's lawyer rescheduled the deposition for August 14. On August 14, the defense lawyer showed up for the deposition in the company of a nine-year-old boy, whom she introduced as the vice president. Upon questioning by the plaintiff's lawyer, the defense lawyer laughingly explained that the young boy was the vice president's son, that he knew nothing about the case, and that the vice president was unable to attend. At that point, the plaintiff's lawyer invited the defense lawyer to accompany him to see the judge to whom the case was assigned. After hearing the story, the judge asked the defense lawyer what day and time the witness would assuredly be available, and the defense lawyer said that August 16 would be good. The judge then entered a formal order that the deposition would be taken that day. The judge also warned that the defense lawyer would be sanctioned if she did not present the witness on August 16, and that the issues of trade secret misappropriation and use would be deemed proven in plaintiff's favor. Alas, on August 16, the defense lawyer again showed up alone, explaining that the vice president had to make an unexpected trip out of state. After appropriate notice and hearing, the judge ordered that the issues of trade secret misappropriation and use would be deemed proven in plaintiff's favor. She also ordered the vice president to pay 60% of the plaintiff's expenses and attorneys' fees incurred because of the failed deposition attempts, and she ordered the defense lawyer to pay the other 40% of plaintiff's expenses and attorneys' fees. Was the judge correct in holding the defense lawyer subject to litigation sanction? Press Enter or Space to submit the answer A Yes, because the defense lawyer violated the court's discovery order by showing up on August 16 without the witness. B Yes, because the facts show that the defense lawyer acted obstinately and disrespectfully in defiance of the judge's direct order. C No, because the facts show that the defense lawyer could not control the witness, and that she was therefore not at fault when he failed to show up on August 16. D No, because when a party or a managing agent of a party violates a discovery order, the sanction must be imposed on the party or managing agent, not on the lawyer.

There are several legal theories on which sanctions could be imposed here, but the most obvious is Rule 37 of the Federal Rules of Civil Procedure, which is specifically designed for discovery abuse. When a party's managing agent fails to show up for a properly scheduled deposition, Rule 37 gives the judge a wide choice of sanctions, including such things as: an order that the issues in question be deemed proven in favor of the innocent party; an order forbidding the offender from offering proof on the issues in question; an order striking the offender's pleadings; an order finding the offender guilty of contempt of court; and an order that the offending party, its lawyer, or both must pay the innocent party's expenses and attorney fees incurred because of the violation. (A) is better than (B) because (B) implies an incorrect legal standard; a lawyer can be sanctioned under Rule 37 even if she did not act obstinately or disrespectfully. (C) and (D) are not as good as (A) because they imply that a lawyer cannot be sanctioned for discovery abuses by her client or her client's managing agent. That is not correct. A lawyer has to walk a thin line between bullying her client and being bullied by her client. The defense lawyer was not blame-free here. She was on notice that the deponent was an uncooperative witness. He had failed to show up three times previously, and the judge had specifically warned the defense lawyer that she would be sanctioned if she showed up without the deponent on August 16. The defense lawyer could have informed the deponent's corporate superior about the situation, seeking to have the superior force the deponent to behave. She could also have reminded her client that she may withdraw as counsel if the deponent does not cooperate [ABA Model Rule 1.16(b)], and that she must withdraw if the deponent's mulishness forces her to violate a court order [ABA Model Rule 1.16(a)]. The defense lawyer's failure to take these or similar steps to assure the deponent's presence makes the sanctions order appropriate.


Set pelajaran terkait

Principles of Management - Ch 9 Test

View Set

First Aid OB GYN Clerkship HY Shelf Review

View Set

CHAPTER FOUR: TAXES, RETIREMENT, AND OTHER INSURANCE CONCEPTS

View Set

Lab 1.2: Networking Topologies and Characteristics

View Set